Integral over an interval of the normal distribution












0












$begingroup$


Assume that $f$ is the normal distribution function with mean equal to $0$ and variance equal to $sigma^2$.



Set some interval $I=[a,b]$ in $mathbb{R}^+$.



Denote the integral of $f$ over $I$ by $S_I(sigma^2)$.



My question: is $S_I(sigma^2)$ monotonous (decreasing?) as $sigma^2$ increases continuously? Or is it dependent on $I$ or the initial value of $sigma^2$?



Thank you!










share|cite|improve this question









$endgroup$

















    0












    $begingroup$


    Assume that $f$ is the normal distribution function with mean equal to $0$ and variance equal to $sigma^2$.



    Set some interval $I=[a,b]$ in $mathbb{R}^+$.



    Denote the integral of $f$ over $I$ by $S_I(sigma^2)$.



    My question: is $S_I(sigma^2)$ monotonous (decreasing?) as $sigma^2$ increases continuously? Or is it dependent on $I$ or the initial value of $sigma^2$?



    Thank you!










    share|cite|improve this question









    $endgroup$















      0












      0








      0





      $begingroup$


      Assume that $f$ is the normal distribution function with mean equal to $0$ and variance equal to $sigma^2$.



      Set some interval $I=[a,b]$ in $mathbb{R}^+$.



      Denote the integral of $f$ over $I$ by $S_I(sigma^2)$.



      My question: is $S_I(sigma^2)$ monotonous (decreasing?) as $sigma^2$ increases continuously? Or is it dependent on $I$ or the initial value of $sigma^2$?



      Thank you!










      share|cite|improve this question









      $endgroup$




      Assume that $f$ is the normal distribution function with mean equal to $0$ and variance equal to $sigma^2$.



      Set some interval $I=[a,b]$ in $mathbb{R}^+$.



      Denote the integral of $f$ over $I$ by $S_I(sigma^2)$.



      My question: is $S_I(sigma^2)$ monotonous (decreasing?) as $sigma^2$ increases continuously? Or is it dependent on $I$ or the initial value of $sigma^2$?



      Thank you!







      probability






      share|cite|improve this question













      share|cite|improve this question











      share|cite|improve this question




      share|cite|improve this question










      asked Jan 16 at 12:07









      DaugmentedDaugmented

      326212




      326212






















          1 Answer
          1






          active

          oldest

          votes


















          1












          $begingroup$

          Consider this plot and the interval around $2$. It's clear that the integral is not monotonous with $sigma$. So in general case, it depends both on interval (for symmetric or wide enough intervals, the integral is monotonous) and on initial value of $sigma^2$ (for each interval there is such $sigma_0$, so for $sigma>sigma_0$ the integral is monotonous).



          enter image description here






          share|cite|improve this answer









          $endgroup$














            Your Answer





            StackExchange.ifUsing("editor", function () {
            return StackExchange.using("mathjaxEditing", function () {
            StackExchange.MarkdownEditor.creationCallbacks.add(function (editor, postfix) {
            StackExchange.mathjaxEditing.prepareWmdForMathJax(editor, postfix, [["$", "$"], ["\\(","\\)"]]);
            });
            });
            }, "mathjax-editing");

            StackExchange.ready(function() {
            var channelOptions = {
            tags: "".split(" "),
            id: "69"
            };
            initTagRenderer("".split(" "), "".split(" "), channelOptions);

            StackExchange.using("externalEditor", function() {
            // Have to fire editor after snippets, if snippets enabled
            if (StackExchange.settings.snippets.snippetsEnabled) {
            StackExchange.using("snippets", function() {
            createEditor();
            });
            }
            else {
            createEditor();
            }
            });

            function createEditor() {
            StackExchange.prepareEditor({
            heartbeatType: 'answer',
            autoActivateHeartbeat: false,
            convertImagesToLinks: true,
            noModals: true,
            showLowRepImageUploadWarning: true,
            reputationToPostImages: 10,
            bindNavPrevention: true,
            postfix: "",
            imageUploader: {
            brandingHtml: "Powered by u003ca class="icon-imgur-white" href="https://imgur.com/"u003eu003c/au003e",
            contentPolicyHtml: "User contributions licensed under u003ca href="https://creativecommons.org/licenses/by-sa/3.0/"u003ecc by-sa 3.0 with attribution requiredu003c/au003e u003ca href="https://stackoverflow.com/legal/content-policy"u003e(content policy)u003c/au003e",
            allowUrls: true
            },
            noCode: true, onDemand: true,
            discardSelector: ".discard-answer"
            ,immediatelyShowMarkdownHelp:true
            });


            }
            });














            draft saved

            draft discarded


















            StackExchange.ready(
            function () {
            StackExchange.openid.initPostLogin('.new-post-login', 'https%3a%2f%2fmath.stackexchange.com%2fquestions%2f3075657%2fintegral-over-an-interval-of-the-normal-distribution%23new-answer', 'question_page');
            }
            );

            Post as a guest















            Required, but never shown

























            1 Answer
            1






            active

            oldest

            votes








            1 Answer
            1






            active

            oldest

            votes









            active

            oldest

            votes






            active

            oldest

            votes









            1












            $begingroup$

            Consider this plot and the interval around $2$. It's clear that the integral is not monotonous with $sigma$. So in general case, it depends both on interval (for symmetric or wide enough intervals, the integral is monotonous) and on initial value of $sigma^2$ (for each interval there is such $sigma_0$, so for $sigma>sigma_0$ the integral is monotonous).



            enter image description here






            share|cite|improve this answer









            $endgroup$


















              1












              $begingroup$

              Consider this plot and the interval around $2$. It's clear that the integral is not monotonous with $sigma$. So in general case, it depends both on interval (for symmetric or wide enough intervals, the integral is monotonous) and on initial value of $sigma^2$ (for each interval there is such $sigma_0$, so for $sigma>sigma_0$ the integral is monotonous).



              enter image description here






              share|cite|improve this answer









              $endgroup$
















                1












                1








                1





                $begingroup$

                Consider this plot and the interval around $2$. It's clear that the integral is not monotonous with $sigma$. So in general case, it depends both on interval (for symmetric or wide enough intervals, the integral is monotonous) and on initial value of $sigma^2$ (for each interval there is such $sigma_0$, so for $sigma>sigma_0$ the integral is monotonous).



                enter image description here






                share|cite|improve this answer









                $endgroup$



                Consider this plot and the interval around $2$. It's clear that the integral is not monotonous with $sigma$. So in general case, it depends both on interval (for symmetric or wide enough intervals, the integral is monotonous) and on initial value of $sigma^2$ (for each interval there is such $sigma_0$, so for $sigma>sigma_0$ the integral is monotonous).



                enter image description here







                share|cite|improve this answer












                share|cite|improve this answer



                share|cite|improve this answer










                answered Jan 16 at 12:37









                Vasily MitchVasily Mitch

                2,6791312




                2,6791312






























                    draft saved

                    draft discarded




















































                    Thanks for contributing an answer to Mathematics Stack Exchange!


                    • Please be sure to answer the question. Provide details and share your research!

                    But avoid



                    • Asking for help, clarification, or responding to other answers.

                    • Making statements based on opinion; back them up with references or personal experience.


                    Use MathJax to format equations. MathJax reference.


                    To learn more, see our tips on writing great answers.




                    draft saved


                    draft discarded














                    StackExchange.ready(
                    function () {
                    StackExchange.openid.initPostLogin('.new-post-login', 'https%3a%2f%2fmath.stackexchange.com%2fquestions%2f3075657%2fintegral-over-an-interval-of-the-normal-distribution%23new-answer', 'question_page');
                    }
                    );

                    Post as a guest















                    Required, but never shown





















































                    Required, but never shown














                    Required, but never shown












                    Required, but never shown







                    Required, but never shown

































                    Required, but never shown














                    Required, but never shown












                    Required, but never shown







                    Required, but never shown







                    Popular posts from this blog

                    Human spaceflight

                    Can not write log (Is /dev/pts mounted?) - openpty in Ubuntu-on-Windows?

                    File:DeusFollowingSea.jpg